LSAT and Law School Admissions Forum

Get expert LSAT preparation and law school admissions advice from PowerScore Test Preparation.

 Administrator
PowerScore Staff
  • PowerScore Staff
  • Posts: 8917
  • Joined: Feb 02, 2011
|
#23655
Complete Question Explanation

Assumption-SN. The correct answer choice is (D)

In this stimulus a typical credit card scenario is presented and then a conclusion drawn. Again, pay careful attention to the conclusion: "customers who purchase merchandise [by mail] spend less" than they would spend if they went to a store. Although the premises demonstrate that the "purchase price" is lower by mail, they never mention the idea of how much the customer actually spends, which is a new element in the conclusion. This demonstrates the weakness of the argument, and where a Defender assumption might be necessary. We are looking for an answer choice that, when we negate it, destroys the conclusion of the argument.


Answer Choice (A): This answer choice, when negated, basically says that other credit-card companies offer the same merchandise at lower prices. This does not affect the conclusion one way or another since it deals only with the comparison between items offered by this particular company by mail and those items in a retail store.

Answer Choice (B): This answer choice, when negated, only says that customers can buy merchandise not available in stores. It does not affect the conclusion because it says nothing about money being spent or prices.

Answer Choice (C): This answer choice, when negated, says the companies make customers accrue a large number of points, but still reinforces the fact that prices are lower. When negated, it does not destroy the conclusion.

Answer Choice (D): This is the correct answer choice. Notice that it deals specifically with the gap in the reasoning – spending. When negated, this answer choice destroys the conclusion because, even though the prices may be lower, the customer spends more when shipping is added.

Thus, this answer choice is necessary for the conclusion to follow.

Answer Choice (E): This answer choice, when negated, does not affect the stimulus in any way. It introduces the new element of the "manufacturers' suggested retail price," which is irrelevant to the argument in the stimulus.
 nmgee
  • Posts: 15
  • Joined: Jun 01, 2018
|
#46489
I correctly selected (D), but did have some trouble choosing between (C) and (D). The conclusion is that “customers who...use bonus points [earned by using the credit card] spend less than they would” if they had purchased at a retail store.

I read (C) as a contender because “requiring a larger number of bonus points before becoming eligible” suggests customers may need to spend more on non-bonus point (non discounted) purchases just to be able to use those bonus points, and thus ultimately might not be spending less than they would have if they’d just purchased the item in stores ($ needed to reach bonus-point-spending-eligibility *could* exceed the customer’s intended expenditures) i.e. spending more on bonus-point-earning-activity could potentially exceed what the person would have paid if they didn’t care achieve bonus-point-spending-eligibility, even if total expenditures included a retail-priced item purchased cheaper through bonus points.

How do I know not to make such an assumption regarding the bonus points?

Thanks!
 Alex Bodaken
PowerScore Staff
  • PowerScore Staff
  • Posts: 136
  • Joined: Feb 21, 2018
|
#46557
nmgee,

Thanks for the question, and it's a good one. I think that the key phrase from the conclusion here is "if they purchased the same merchandise," which indicates that the conclusion is drawing a direct comparison between the bonus-point spending on certain merchandise and retail spending on those same items (I would prefer the phrase "if and when..." to more clearly make this comparison, but the "if" is sufficient, even if it is a little unclear). I.E., this clause is meant to exclude consideration of other spending, including whatever spending may have gone on prior in accumulating said bonus points. I agree that it is a somewhat difficult distinction to make, but again, I think that phrase is tailored narrowly enough to exclude other purchases, effectively eliminating (C) from contention.

Hope that helps!
Alex
 LSATLondon
  • Posts: 4
  • Joined: Jul 03, 2019
|
#68375
Can you explain how you would diagram this SN relationship? I'm confused as I didn't identify it as a sufficient-necessary relationship

Thank you!
 James Finch
PowerScore Staff
  • PowerScore Staff
  • Posts: 943
  • Joined: Sep 06, 2017
|
#68384
Hi LSAT London,

The conditional relationship given is the conclusion--it's not a certainty, but rather set off by the "if" in the conclusion. That said, it's not worth actually diagramming this relationship, as the key logical gap in the stimulus is the comparison between total spending if bought in the credit card catalog versus total spending in stores. We're given several components of this relationship, but not the total number in either instance. So the best Prephrase would be one that partly addresses this cost issue, making the catalog price lower than stores, such as shipping, tax, sale prices, etc., as (D) does (shipping costs).

Hope this helps!
User avatar
 Esquire123
  • Posts: 15
  • Joined: Jan 25, 2023
|
#99317
Administrator wrote: Fri Apr 29, 2016 1:47 pm Complete Question Explanation

Assumption-SN. The correct answer choice is (D)

In this stimulus a typical credit card scenario is presented and then a conclusion drawn. Again, pay careful attention to the conclusion: "customers who purchase merchandise [by mail] spend less" than they would spend if they went to a store. Although the premises demonstrate that the "purchase price" is lower by mail, they never mention the idea of how much the customer actually spends, which is a new element in the conclusion. This demonstrates the weakness of the argument, and where a Defender assumption might be necessary. We are looking for an answer choice that, when we negate it, destroys the conclusion of the argument.


Answer Choice (A): This answer choice, when negated, basically says that other credit-card companies offer the same merchandise at lower prices. This does not affect the conclusion one way or another since it deals only with the comparison between items offered by this particular company by mail and those items in a retail store.

Answer Choice (B): This answer choice, when negated, only says that customers can buy merchandise not available in stores. It does not affect the conclusion because it says nothing about money being spent or prices.

Answer Choice (C): This answer choice, when negated, says the companies make customers accrue a large number of points, but still reinforces the fact that prices are lower. When negated, it does not destroy the conclusion.

Answer Choice (D): This is the correct answer choice. Notice that it deals specifically with the gap in the reasoning – spending. When negated, this answer choice destroys the conclusion because, even though the prices may be lower, the customer spends more when shipping is added.

Thus, this answer choice is necessary for the conclusion to follow.

Answer Choice (E): This answer choice, when negated, does not affect the stimulus in any way. It introduces the new element of the "manufacturers' suggested retail price," which is irrelevant to the argument in the stimulus.
Hi there,

I am still having trouble eliminating answer choice E. The stimulus does bring up the idea of manufactures' suggested retail price in sentence two, so answer choice E bringing it back up seemed relevant. If anything, I was hesitant about answer choice E because it seemed to be a repetition of one of the premises. Also, I'm struggling to figure out what the negation of this statement would sound like. What's the negation of "frequently."

I'm also having trouble understanding answer choice D. I'm really bad with math, so that might be the reason I can't wrap my head around how this is the correct answer. I eliminated answer choice D because of the wording "does not increase the amount customers spend to an amount grater than they would spend..." My understanding of this phrasing was that if the manufactures' suggested retail price for a designer bag is $1,000, then the credit card bonus program cannot total to $1000.01 (or higher) once shipping is accounted for. However, to me this still leaves open the possibility that this designer bag can end up costing exactly $1,000 through the credit card program. If this is the case, the credit card holder isn't spending less than they would spend if they purchased the same merchandise in a retail store. This answer choice seemed to weaken the argument and therefore I quickly eliminated it. Clearly, I misunderstood something along the way. Any help trying to figure out where I went wrong is much appreciated:)
User avatar
 Jeff Wren
PowerScore Staff
  • PowerScore Staff
  • Posts: 389
  • Joined: Oct 19, 2022
|
#99351
Hi Esquire123,

The first thing to be aware of is that this question is an Assumption question (sometimes called a Necessary Assumption question). I mention this because, as a question type, they often give people a lot of difficulty and people often aren't clear on what exactly they are looking for in an answer.

What a correct answer to an Assumption question contains is a statement that whoever is making the argument has to believe in order for their argument to make sense. The best way to determine which answer contains such a statement is to use the Assumption Negation Technique. More information on this technique is found in Lesson 5 of the PowerScore LSAT Course and Chapter 11 of The Logical Reasoning Bible.

When you negate Answer D, the negated answer reads "The amount credit-card customers pay for shipping the merchandise ordered by mail does increase the amount customers spend to an amount greater than they would spend if they purchased the same merchandise in retail stores." If that statement were true, then the argument completely falls apart because it would mean that the shipping costs outweigh the savings that these customers would have received, which directly contradicts the conclusion. Because the negated answer destroys the argument, the original (non-negated) answer is necessary to the argument and is therefore the correct answer.

While it is also true that the author assumes that the shipping costs won't be exactly equal to the savings, don't get tripped up on that because that doesn't make this answer wrong. There can be (and often are) many assumptions that are necessary for an argument.

As for Answer E, the easiest way to negate the answer would be to just add the word "not" before "frequently."

"The merchandise available to the company’s credit-card customers using the bonus points is not frequently sold in retail stores at prices that are higher than the manufacturers’ suggested retail prices."

You could also use synonyms for "not frequently" such as "infrequently" or even "rarely" (although "rarely" may be a bit stronger than "infrequently," it gets at the idea closely enough here.) The reason that this negated statement doesn't hurt the argument is that it is completely fine if all of the retail stores sell the products at exactly the manufacturers' suggested retail prices, which is completely consistent with rarely selling it higher than suggested retail price. In other words, the comparison in the argument is between the credit card bonus purchases via mail and retail stores. We are told that the credit card bonus point purchases are at a lower price than the manufacturers' suggested retail prices, so as long as the retail stores sell them at those suggested prices, the argument is fine. The retailers don't have to sell the items above the retail price, they just need to not sell them below the retail price (such as having a sale) because that would be a problem for the argument.

Get the most out of your LSAT Prep Plus subscription.

Analyze and track your performance with our Testing and Analytics Package.